Answered You can hire a professional tutor to get the answer.

QUESTION

Suppose L is a regular language, and M = (Q, , , q0, A) is a deterministic finite state machine such that L(M) = L. Prove that if |Q| = 2 then one of...

  1. Suppose L is a regular language, and M = (Q, Σ, δ, q0, A) is a deterministic finite state machine such that L(M) = L. Prove that if |Q| = 2 then one of the following hold: (i)L=∅(ii)ε∈L,or(iii)∃a∈Σsuch that a∈L. 
Show more
LEARN MORE EFFECTIVELY AND GET BETTER GRADES!
Ask a Question